heyyyyyyyyyyyyyyyyyyy

Answers

Answer 1

Let the length of tape required be x centimeter

15centimeters to rap 5 present

x centimeters will rap 6 present

[tex]\begin{gathered} 15\operatorname{cm}=5 \\ x\text{ cm=6} \\ \text{hence} \\ 5x=15\times6 \\ 5x=90 \\ \text{divide both side by 5, we have} \\ x=\frac{90}{5} \\ x=18\operatorname{cm} \end{gathered}[/tex]

H


Related Questions

A firm has a monthly fixed cost of $2000, and the variable cost per unit of its product is $25. a. Determine the cost function. b. The revenue R obtained by selling x units is given by R(x) = 60x - 0.01x2. Determine the number of units that must be sold each month so as to maximize the revenue. What is the maximum revenue? c. How many units must be produced and sold each month to obtain a maximum profit? What is the maximum profit?

Answers

a) We can write the cost function (in function of the units made) as the sum of the fixed cost (2000) and the variable cost (25*x):

[tex]C(x)=2000+25x[/tex]

b) The revenue R(x) is:

[tex]R(x)=60x-0.01x^2[/tex]

To find the value of x that maximizes R(x) we derive R(x) and equal it to 0:

[tex]\frac{dR}{dx}=60(1)-0.01(2x)=60-0.02x[/tex][tex]\begin{gathered} \frac{dR}{dx}=0 \\ 60-0.02x=0 \\ 60=0.02x \\ x=\frac{60}{0.02} \\ x=3000 \end{gathered}[/tex]

We can now calculate the maximum revenue as R(3000):

[tex]\begin{gathered} R(3000)=60\cdot3000-0.01\cdot(3000)^2 \\ R(3000)=180000-0.01\cdot9000000 \\ R(3000)=180000-90000 \\ R(3000)=90000 \end{gathered}[/tex]

c) The profit function P(x) can be calculated as the difference between the revenue and the cost:

[tex]\begin{gathered} P(x)=R(x)-C(x) \\ P(x)=(60x-0.01x^2)-(2000+25x) \\ P(x)=-0.01x^2+60x-25x-2000 \\ P(x)=-0.01x^2+35x-2000 \end{gathered}[/tex]

In the same way as we did in b), we can calculate the number of units x that maximize the profit by deriving P(x) and making it equal to 0:

[tex]\begin{gathered} \frac{dP}{dx}=-0.01(2x)+35(1)-2000(0)=0 \\ -0.02x+35=0 \\ 35=0.02x \\ x=\frac{35}{0.02} \\ x=1750 \end{gathered}[/tex]

The maximum profit can be then calculated as P(1750):

[tex]\begin{gathered} P(1750)=-0.01(1750)^2+35(1750)-2000 \\ P(1750)=-0.01\cdot3062500+61250-2000 \\ P(1750)=-30625+61250-2000 \\ P(1750)=28625 \end{gathered}[/tex]

We can graph R(x) and P(x) as:

Answer:

a) C(x) = 2000 + 25x

b) x = 3000 units

R(3000) = $90000

c) x = 1750 units

P(1750) = $28625

Find the equation of line containing given points. Write the equation in slope- intercept form (0,2)(2,-3)

Answers

Answer:

[tex]y=\frac{-5x}{2}\text{ + 2}[/tex]

Explanation:

Here, we want to get the equation of the line

The general equation of a line in slope-intercept form is:

[tex]y\text{ = mx + b}[/tex]

where m is the slope and b is the y-intercept

We can get the equation through the following:

[tex]\frac{y-y_1}{x-x_1}\text{ = }\frac{y_2-y_1}{x_2-x_1}[/tex]

where (x1,y1) is (0,2) and (x2,y2) is (2,-3)

Substituting the values, we have it that:

[tex]\begin{gathered} \frac{y-2}{x-0}\text{ = }\frac{-3-2}{2-0} \\ \\ \frac{y-2}{x}\text{ = }\frac{-5}{2} \\ \\ \left(y-2\right)\text{ = }\frac{-5x}{2} \\ \\ y\text{ = }\frac{-5x}{2}\text{ + 2} \end{gathered}[/tex]

determine the miss term 3:5:7:?:25:

Answers

7x5=(35) is the missing term

Determine algebraically ifthe function is even, odd, or neither. f(x)= -9

Answers

Background:

• Even ,functions are symmetric with respect to the ,y-axis,.

,

• Odd ,functions are symmetric with respect to the origin.

,

• Neither ,has no symmetry with respect to the origin or ,y-axis,.

Answer: Even

Out of 40 students, 14 are taking English composition, 29 are taking Chemistry. If 5 students are in both class how many students are in neither classes.What is the probability that a randomly chosen student from this group is taking only chemistry class.

Answers

The probability that a randomly chosen student from this group is taking only a chemistry class will be 0.60.

What is probability?

Its fundamental concept is that someone will nearly surely occur. The proportion of positive events in comparison to the total of occurrences.

Then the probability is given as,

P = (Favorable event) / (Total event)

Out of 40 understudies, 14 are taking English organization, and 29 are taking chemistry. Assuming 5 understudies are in both classes the number of understudies that are in neither class.

The number of students in neither class will be given as,

⇒ 40 - (29 + 14 - 5)

⇒ 40 - 38

⇒ 2

The probability that a randomly chosen student from this group is taking only a chemistry class will be

P = (29 - 5) / 40

P = 24 / 40

P = 0.60

The probability that a randomly chosen student from this group is taking only a chemistry class will be 0.60.

More about the probability link is given below.

https://brainly.com/question/795909

#SPJ1

find the coordinates of the ends of each latus rectum and equations of asymptotes.

Answers

For conic section of the form:

[tex](\frac{x^2}{a^2})-(\frac{y^2}{b^2})=1[/tex]

The Ends of the Lactus Rectum is given as:

[tex]L=(ae,\frac{b^2}{a}),L=(ae,\frac{-b^2}{a})[/tex]

The e in the equation above is the Eccentricity of the Hyperbola.

This can be obtained by the formula:

[tex]e=\frac{\sqrt[]{a^2+b^2}}{a}[/tex]

Thus, comparing the standard form of the conic with the given equation, we have:

[tex]\begin{gathered} \frac{(y+8)^2}{16}-\frac{(x-3)^2}{9}=1 \\ \text{This can be further expressed in the form:} \\ \frac{(y+8)^2}{4^2}-\frac{(x-3)^2}{3^2}=1 \\ By\text{ comparing this with:} \\ \frac{x^2}{a^2}-\frac{y^2}{b^2}=1 \\ We\text{ can deduce that:} \\ a=4;b=3 \end{gathered}[/tex]

Then, we need to obtain the value of the Eccentiricity, e.

[tex]\begin{gathered} e=\frac{\sqrt[]{a^2+b^2}}{a} \\ e=\frac{\sqrt[]{4^2+3^2}}{4} \\ e=\frac{\sqrt[]{16+9}}{4} \\ e=\frac{\sqrt[]{25}}{4}=\frac{5}{4} \end{gathered}[/tex]

Hence, the coordinate of the ends of the each lactus rectum is:

[tex]\begin{gathered} L=(ae,\frac{b^2}{a}),L=(ae,\frac{-b^2}{a}_{}) \\ L=(4\times\frac{5}{4},\frac{3^2}{4}),L=(4\times\frac{5}{4},\frac{-3^2}{4}) \\ L=(5,\frac{9}{4}),L=(5,\frac{9}{4}) \end{gathered}[/tex]

help meeeeeeeeeeeeeee pleaseeeeeee

Answers

Answer: 9.7 seconds

Step-by-step explanation:

[tex]16t^2 =1503\\\\t^2 =1503/16\\\\t=\sqrt{1503/16} \text{ } (t > 0)\\\\t \approx 9.7[/tex]

pls help me wi this question

Answers

Answer:

1 block west and 8 blocks north

Step-by-step explanation:

One block east and two blocks north to the coffee shop. Subtract two blocks west from the one block east, and you get one block west. Add the six blocks north to the two blocks north, and you get eight blocks north.

if a/ b+1 = 2, what does 2b equal?

Answers

[tex]\frac{a}{b+1}=2[/tex]

The question and the Triangle is in the same image. I'm on point 3

Answers

ANSWER

• Ratio: 2

,

• Lengths of the sides of the new triangle: 6, 6√2

,

• Circles: ,see explanation

,

• Triangle: ,see explanation

EXPLANATION

• The ratio is the quotient between the length of segment DE and, for example, segment AB,

[tex]\frac{DE}{AB}=\frac{6}{3}=2[/tex]

• Now, we have to multiply each side of triangle ABC by 2 to obtain the lengths of the sides of triangle DEF,

[tex]\begin{cases}DF=2BC=2\cdot3=6 \\ \\ EF=2AC=2\cdot\sqrt[]{3^2+3^2}=2\cdot\sqrt[]{2\cdot9}=6\sqrt[]{2}\end{cases}[/tex]

• Next, draw the two circles mentioned,

• The intersection between the two circles is point F. The ,triangle is,

Gianna purchased 5\tfrac{1}{2}5
2
1

pints of ice cream for a party. If each guest will be served exactly \tfrac{1}{3}
3
1

pint of ice cream, what is the maximum number of guests Gianna can serve?

Answers

The maximum number of guests Gianna can serve will be 16.

What is a fraction?

A fraction is simply a piece of a whole. The number is represented mathematically as a quotient where the numerator and denominator are split. In a simple fraction, the numerator as well as the denominator are both integers

From the information, Gianna purchased 5 1/2 pints of ice cream for a party. If each guest will be served exactly 1/3 pint of ice cream

The maximum number of guests Gianna can serve will be:

= 5 1/2 ÷ 1/3

= 11/2 ÷ 1/3

= 11/2 × 3

= 33/2

= 16.5

The maximum number will be 16.

Learn more about fractions on:

brainly.com/question/78672

#SPJ1

Complete question

Gianna purchased 5 1/2 pints of ice cream for a party. If each guest will be served exactly 1/3 pint of ice cream, what is the maximum number of guests Gianna can serve?

Find the domain of the piecewise function and evaluate it for the given values. Then use the drop down menu to select the correct symbols to indicate your answer in interval notation. If a number is not an integer then round it to the nearest hundredth. To indicate positive infinifty ( \infty ) type the three letters "inf". To indicate negative infinity(-\infty ) type "-inf" with no spaces between characters. f(x) = \left\lbrace \begin{array}{cc} x+1 & x<-2 \\ -2x-3 & x\ge -2 \end{array}\right. The domain is:AnswerAnswer,AnswerAnswerf(-3)=Answerf(-2)=Answerf(-1)=Answerf(0)=Answer

Answers

SOLUTION

Let us make a graph of the function

[tex]\begin{gathered} f(x)=x+1\mleft\{x<-2\mright\} \\ f(x)=-2x-3\mleft\{x\ge-2\mright\} \end{gathered}[/tex]

This is shown below

(a) The domain is usually determined from the x-axis. From the graph, the domain is all real numbers, that is, it is infinite for both positive and negative values for x, as you can see that if the graph is extended, there is no limit for x-values that it can contain. So the domain is (-infinity, infinity)

Hence the answer, Domain is

(-inf, inf)

(b)

[tex]f(-3)[/tex]

In

[tex]\begin{gathered} f(x)=x+1\{x<-2\} \\ -3\text{ is less than -2, so -3 is valid here, so } \\ f(-3)=-3+1 \\ f(-3)=-2 \end{gathered}[/tex]

Hence, the answer is -2

(c)

[tex]f(-2)[/tex]

In

[tex]\begin{gathered} f(x)=-2x-3\{x\ge-2\} \\ -2\text{ is equal to -2, so we will put -2 for x here, so } \\ f(-2)=-2(-2)-3 \\ f(-2)=4-3 \\ f(-2)=1 \end{gathered}[/tex]

Hence, the answer is 1

(d)

[tex]f(-1)[/tex]

In

[tex]\begin{gathered} f(x)=-2x-3\{x\ge-2\} \\ -1\text{ is greater than -2, hence, it is valid, so we will put -1 for x here, so } \\ f(-1)=-2(-1)-3 \\ f(-1)=2-3 \\ f(-1)=-1 \end{gathered}[/tex]

Hence, the answer is -1

(e)

[tex]f(0)[/tex]

In

[tex]\begin{gathered} f(x)=-2x-3\{x\ge-2\} \\ 0\text{ is greater than -2, so we will put 0 for x here, so } \\ f(0)=-2(0)-3 \\ f(0)=0-3 \\ f(0)=-3 \end{gathered}[/tex]

Hence, the answer is -3

Write an equation in slope-intercept form for the line that passes through (0,4) and is parallel to the line described by y=3x-7

Answers

slope-intercept form of a line (L1): y = mx + b; where m is the slope and b the y - intercept.

If two lines are parallel, that means the slope of both equations are equal.

L2: y = 3x - 7, m = 3

So, m for L1 is m = 3.

Now we just need to find y - intercept, as follows:

L1 passes through (0,4), then:

4 = 3(0) + b

4 = 0 + b

b = 4

The equation for the line is y = 3x + 4

Please help me with this problem it’s 1 question of 3 please help me I will leave good feedback for you(((

Answers

tan T = 8/8 = 1

tan ^-1 T = 45°

angle T = 45°

2) angle J = tan J = 2/4

tan J = 1/2

tan^-1 J = 26.57

rounded J = 26.6

Final answer is J= 26.6

Andrea labels ten cards with the numbers 1 through 10. She places the cards with prime numbers (2,3,5,7) in one box, and places the rest of the cards in another box. If Andrea draws one random card from the box of prime numbers and then one random card from the other box, how many different pairs of numbers are possible outcomes?

Answers

Box with prime numbers = 2, 3, 5, 7

Box with the other numbers = 1, 4,6,8,9, 10

Possible pairs

12 13 15 17 Four pairs

42 43 45 47 Four pairs

62 63 65 67 Four pairs

82 83 85 87 Four pairs

92 9 3 95 97 Four pairs

102 103 105 107 Four pairs

Total numbers of pairs 24 pairs

A fish descends 4 meters per minute for 2 minutes. Then it ascends 3 meters per minute for 3 minutes. What is the total distance, in meters, the fish traveled? ​

Answers

Answer:

1 meter of dispalcement,

17 meters of path distance

Step-by-step explanation:

-4meters/min * 2min = -8meter

3 m/m * 3m = 9meters

-8+9 = 1 meter of displacement, it's looking for the total path's distance, then that is just |-8|+|9|=17 meters

I hope this helps

Answer:

The fish ascended a total of meter.

Step-by-step explanation:

distance= rate x time

D= -4(2)= -8m

A= 3(3)= 9m

-8m+9m= 1m

;)

3. A car originally cost $16,000. The owner reduced the price of the car by 20%. After a few
weeks, the owner reduced the price of the car by another 20%, Belinda then purchased the
If a 3% sales tax was added, how much did Belinda pay for the car?
A. $9,600.80
B. $9,888,20
C. $10,240.00
D. $10,547.20

Answers

Answer:

the answer is D: $10547.20

The sale price of women's wool coats is $ ____question attached below

Answers

By '70% off' the text means that there is a discount of 70% on the item, in other words we are only paying 30% of the original value of the item. Therefore if we multiply 30% by the value of the item, we will find the price. This is done below:

[tex]\begin{gathered} \text{price}=220\cdot30\text{ \%} \\ \text{price}=220\cdot\frac{30}{100} \\ \text{price}=220\cdot0.3 \\ \text{price}=66 \end{gathered}[/tex]

The price of the with discount coat is $66

Proof Practice

Given: ∠ PQR and ∠ XYZ are complementary
m ∠ XYZ = 10 degrees

Prove: m ∠ PQR = 80 degrees

5 statements and Reasons needed

Answers

Answer:

1. Given (angle PQR and angle XYZ are complimentary)

2. Definition of complimentary angles (Complimentary means they are added to 90 degrees)

3. Definition of Complimentary angles (Complimentary angles only include two angles)

4. Subtraction postulate (90 degrees - 10 degrees= 80 degrees which proves they are complimentary)

5. Complimentary angles theorem (angle XYZ is 80 degrees)

Sorry if that's too vague, but I really hope this helps you! I bet someone elses answer would clear it up better, so more research or finding a different answer would probably help, just to confirm. I tried to explain as much as I could, I'm not good at teaching what I know.

I’m not sure how to solve 3d. College calculus 1

Answers

Step 1

Given;

[tex]f(x)=(x^2+5)^3[/tex]

Required; To simplify

[tex]\frac{f(x)-f(0)}{x},\text{ x}\ne0[/tex]

Step 2

[tex]\frac{(x^2+5)^3-(0^2+5)^3}{x}[/tex][tex]\mleft(a+b\mright)^3=a^3+3a^2b+3ab^2+b^3---(apply\text{ p}\operatorname{erf}ect\text{ cube formula)}[/tex][tex](x^2+5)^3=(x^2)^3+3(x^2)^2(5)+3x^2(5^2)+5^3[/tex][tex](x^2+5)^3=x^6+15x^4+75x^2+125[/tex][tex]\frac{(x^6+15x^4+75x^2+125)-125}{x}[/tex][tex]\begin{gathered} \frac{x^6+15x^4+75x^2+125-125}{x} \\ \frac{x^6+15x^4+75x^2}{x} \\ \frac{f(x)-f(0)}{x}=x^5+15x^3+75x \end{gathered}[/tex]

Hence if we factorize we get;

[tex]x(x^4+15x^2+75)[/tex]

Therefore;

[tex]\frac{f(x)-f(0)}{x}=x(x^4+15x^2+75)[/tex]

Answer:

See below

Step-by-step explanation:

f(x) = ( x^2 + 5)^3      f(0) =  5^3 = 125

(x^2+5)^3 = x^6 + 15x^4 + 75 x^2 + 125

so you have       (x^6 + 15x^4 + 75x^2  + 125 - 125) /x

                   = x^5 + 15x^3 + 75x     =   x ( x^4 + 15x^2 + 75)  

how do I find a triangle congruence postulate that can be used to prove that the triangles are congruent if any with the options below?

Answers

First figure

we have that the triangles

ABC is congruent with triangle DCB by HL (hypotenuse leg)

Second figure

The triangles are congruent by Angle-Side- Angle

Third figure

The triangles are congruent by Side-angle -side

y=-x^2-2x+3 what is the vertex of the graph? Answer an order pair.

Answers

Given:

[tex]y=-x^2-2x+3[/tex]

a) To find the vertex:

Here, a=-1, b=-2, and c=3

We know that the formula to find the x- coordinate of the vertex is given by,

[tex]\begin{gathered} -\frac{b}{2a}=-\frac{(-2)}{2(-1)} \\ =-1 \end{gathered}[/tex]

Substitute x=-1 in the given equation we get,

[tex]\begin{gathered} y=-(-1)^2-2(-1)+3 \\ =-1+2+3 \\ =4 \end{gathered}[/tex]

Hence, the vertex of the graph is (-1, 4).

b) To find the range of the graph:

Let us find the y-intercept.

Put x=0, we get

[tex]\begin{gathered} y=-(0)^2-2(0)+3 \\ =3 \end{gathered}[/tex]

From the figure, we observe that

The range of the graph is

[tex]\lbrack0,4\rbrack[/tex]

c) To find the domain of the graph:

Let us find the x-intercept.

Put y=0, we get

[tex]\begin{gathered} -x^2-2x+3=0 \\ (x+3)(x-1)=0 \\ x=-3,1 \end{gathered}[/tex]

From the figure, we observe that,

The domain of the graph is,

[tex]\lbrack-3,0)[/tex]

the circle graph shows how a family budgets its annual income. if the total annual income is $90,000 what amount is budgeted for clothing?

Answers

Answer:

$11700

Step-by-step explanation:

comment if u need the explanantion ;D

Kyle can was the car in 30 minutes. Michael can wash the car in 40 minutes. Working together, can they wash the car in less than 16 minutes?

Answers

No, they can not wash the car in less than 16 minutes while working together.

Given, Kyle can wash the car in 30 minutes.

Michael can wash the car in 40 minutes.

Now, we are asked that working together, can they wash the car in less than 16 minutes.

So, Kyle wash the car = 30 min

Kyle can wash (1/30)th part of the car in 1 min.

Michael wash the car = 40 min

Michael can wash (1/40)th part of the car in 1 min.

Kyle and Michael can together wash the ( 1/30 + 1/40 )th part of the car in 1 min.

Kyle and Michael = 1/30 + 1/40

Kyle and Michael = (4 + 3)/120

Kyle and Michael = 7/120

So, Kyle and Michael can together wash the 7/120th part of the car in 1 min.

and both can together wash the car in 120/7 min i.e. 17.14 min

So, working together both can wash the car in 17.14 min.

Hence, No, they can not wash the car in less than 16 minutes while working together.

Learn more about Direct and Inverse Proportion here https://brainly.com/question/1266676

#SPJ1

pls help I was taught this concept today and I can't understand or get it right!! Find the measure of the arc or angle indicatedFind the measure of mPSQa) 53°b) 248°c) 72°d) 65°

Answers

Step 1:

Find the value of x

Using secant theorem

[tex]<\text{PQR =}\frac{1}{2}\text{ }\timesStep 2:

If x = 9

Then 6x+2 = 6(9) + 2 = 54+2 = 56 degrees

The measure of arc PQ = 14(9) - 14 = 126 - 14 = 112 degrees

Step 3:

The total angle in a circumference is 360 degrees

Therefore ,

mPQ + mPSQ = 360

mPSQ = 360 - mPQ

mPSQ= 360 - 112 = 248degrees

The answer is option B

start new workings.
What value of x makes the two expressions
below equal? Give your answer as a
decimal.
5x-8
First expression
3x+5
Second expression give ur answer as a decimal maths

Answers

The value of x that makes both expressions equal is 6.5.

What are expressions?An expression, often known as a mathematical expression, is a finite collection of symbols that are well-formed in accordance with context-dependent principles.As an illustration, the phrase x + y is one where x and y are terms with an addition operator in between. There are two sorts of expressions in mathematics: numerical expressions, which only contain numbers, and algebraic expressions, which also include variables.

So, the value of x that makes both the expression equal.

So, we can write the expression as:

5x - 8 = 3x + 5

Now, solve for x as follows:

5x - 8 = 3x + 5

5x - 3x = 5 + 8

2x = 13

x = 13/2

x = 6.5

Therefore, the value of x that makes both expressions equal is 6.5.

Know more about expressions here:

https://brainly.com/question/28934492

#SPJ1

find the slope of 4x + 3y equals to

Answers

find the slope of 4x + 3y equals to​

I will assume a value of a

so

4x+3y=a

Isolate the variable y

3y=-4x+a

y=-(4/3)x+a/3

therefore

the slope is -4/3

Note: the value of the slope not depend of the value of a

Translate and solve. Twice a number plus 6 is -14

Answers

translation 2x + 6 = -14

2x = -14 - 6

2x = -20

x = -20/2

x = -10

Please I need you to help me and show the steps please so I can understand it better
I will mark brainliest

Answers

Answer:

LM = √34

Step-by-step explanation:

L is at (-7, 4), and M is at (-2, 1).

LM

[tex] \sqrt{ {( - 7 - ( - 2))}^{2} - {(4 - 1)}^{2} } [/tex]

[tex] \sqrt{ {( - 5)}^{2} + {3}^{2} } = \sqrt{25 + 9} = \sqrt{34} [/tex]

pls help mepls pls pls pls pls pls

Answers

The expression with a coefficient of 10 and a constant of 5 is given by 10x + 5.

What is an equation? What is a coefficient?

An equation is a mathematical statement with an 'equal to' symbol between two expressions that have equal values. In a equation say : ax + b, [a] is called coefficient of [x] and [b] is independent of [x] and hence is called constant.

We have a expression with a coefficient of 10 and a constant of 5.

The expression with a coefficient of 10 and a constant of 5 is given by -

10x + 5

Therefore, the expression with a coefficient of 10 and a constant of 5 is given by 10x + 5.

To solve more questions on equation solving, visit the link below-

brainly.com/question/18067015

#SPJ1

Other Questions
Every surface of the block shown will be painted, except for one of the bases. How many square units will be painted? 3.2 cm 3.2 cm 3.2 cm Examine the following graph of the system of inequalities yx24x3 and y A food worker has prepared alarge pot of rice that must becooled.How should the food workercool the rice safely?a. Place the pot in the refrigeratorb. Cover the pot and leave it at roomtemperaturec. Separate the rice into smaller portionsand refrigerate A girl has scored 72, 76, 74, and 75 on her algebra tests.a. Use an inequality to find the score she must make on the final test to pass the course with an average of 78 or higher, given that the final exam counts three testsb. Explain the meaning of the answer to part (a) Which of these two quotes would look better in an essay?In the rising action, She stood by the window and looked out dully, O. Henry wrote, Tomorrow would be Christmas Day, and she had only $1.87 with which to buy Jim a present, (4).In the rising action, O. Henry writes, She stood by the window and looked out dully, O. Henry wrote, Tomorrow would be Christmas Day, and she had only $1.87 with which to buy Jim a present, (4). Directions read the sentences. information my sister is the captain of her college swim team. consequently, she spends a lot of time training and encouraging her teammates. question which is the correct way to combine the two sentences? answer options with 4 options 1. my sister is the captain of her college swim team; consequently, she spends a lot of time training and encouraging her teammates. 2. my sister is the captain of her college swim team consequently, she spends a lot of time training and encouraging her teammates. 3. my sister is the captain of her college swim team, consequently, she spends a lot of time training and encouraging her teammates. 4. my sister is the captain of her college swim team: consequently, she spends a lot of time training and encouraging her teammates. 1.At 8:30am, Student Life had served 37 meals at their pancake breakfast. By 10:15am, thetotal served had reached 77. Find the serving rate, in number of meals per minute. Keep youranswer as a reduced fraction. Question 13Triesremaining: 3Check for UnderstandingPoints out of2.00Write the equation of the line with a slope of 10 that goes through the point(8,-2) in Slope-Intercept and Point-Slope Form.P FlagquestionSlope-Intercept Form:Point-Slope Form:CheckPrevious page 4x-6x + 15 - x -4 how do I simply this Use the graph below to find the slope of the line. Explain your steps with sentences. What does the circulatory system and ribosomes have in common or do they not have anything in common mishkin, ungerleider, and macko (1983) argue that a key unresolved issue is how information from the dorsal and ventral streams is reintegrated by the brain. they offer a hypothesis about this. what is it? (1-3 sentences) What are abrupt repetitions that occur in changes frequently with no set pattern? -Regular rhythm -Staccato Rhythm -Irregular Rhythm -Progressive Rhythm Part AWhat is the mass in grams of 17.96 mL of acetone?Express your answer to four significant figures and include the appropriate units.mass = Value and UnitsPart BWhat is the volume in milliliters of 7.40 g of acetone?Express your answer to three significant figures and include the appropriate units.V = Value and Units The weather map shows some conditions in the atmosphere at noon on aparticular day.10161016-19201008100410009961024H1012 1008 1008 1012 1016Where would you expect the warmest weather?O A. In the southeastB. In the southwestC. In the northeastD. In the northwest determine the numbers of solutions that exist to the equation below 8 (j - 4) = 2(4j - 16) what type of system enables physicians to place orders electronically, with the order transmitted directly to the recipient? 5.-4h + 3 + 7h 9h - 21 - (True/False) If there is no friction, the efficiency of a machine can be greater than 100%. a motorcycle traveling 100m/s drives off a horizontal ramp and lands a horizontal distance of 40m away from the edge of the ramp, what is the height of the ramp?